0 Daumen
809 Aufrufe

Es seien \( V = \mathbb{R^3} \) und \( T = \lbrace{t_1,t_2,t_3 \rbrace} \) die Basis mit

\( t_1 = (1, -1, 1), \quad t_2 = (0, -1, 1), \quad t_3 = (0, 1, 1) \)

und \( f: V \to V \) die lineare Abbildung mit \( f(t_1) = 2t_1 + t_2, \quad f(t_2) = t_2 - 2t_3, \quad f(t_3) = t_1 + t_2 + t_3 \)

Dass T eine Basis ist, darf als bewiesen verwendet werden.

a)Bestimmen Sie \(D_T(f) \). Da ich diese Schreibweise nicht kenne, gehe ich von \( D_T^{T}(f) \) aus?

Dann ist \(D_T(f) = \begin{pmatrix} 2 && 0 && 1\\ 1 && 1 && 1\\ 0 && -2 && 1 \end{pmatrix}  \)

Das kann ich ja aus der Darstellung von f einfach ablesen.

b) Bestimmen Sie  \(D_T(f^2) \). Hier bin ich mir nicht ganz sicher, ob mein Vorgehen richtig ist:

\( f^2(t_1) = f(f(t_1)) = f(f(\begin{pmatrix} 1 \\ -1 \\ 1\end{pmatrix})) = f(2t_1 + t_2) \)

Nun multipliziere ich \(\begin{pmatrix} 2 \\ 1 \\ 0\end{pmatrix} \) mit der berechneten Darstellungsmatrix aus a)

\( \begin{pmatrix} 2 && 0 && 1\\ 1 && 1 && 1\\ 0 && -2 && 1 \end{pmatrix} \cdot \begin{pmatrix} 2 \\ 1 \\ 0\end{pmatrix}= \begin{pmatrix} 4 \\ 3 \\ -2\end{pmatrix}  \)

Und damit ist \( f(2t_1+ t_2) = f(\begin{pmatrix} 2 \\ -3 \\ 3\end{pmatrix} = 4t_1 + 3t_2 - 2t_3 = \begin{pmatrix} 4 \\ -9 \\ 5\end{pmatrix} \)

Und das ist dann die erste Spalte von  \(D_T(f^2) \). Wenn das bis hierhin soweit richtig ist, muss ich dasselbe ja noch mit \(t_2\) und \(t_3 \)machen. Ist es denn richtig??

Avatar von

1 Antwort

+1 Daumen
 
Beste Antwort

d.h.

\( \small\tau D\tau \, :=  \, \left(\begin{array}{rrr}2&0&1\\1&1&1\\0&-2&1\\\end{array}\right) \)


\( \small f^2:\tau D\tau^{2}= \,\left(\begin{array}{rrr}4&-2&3\\3&-1&3\\-2&-4&-1\\\end{array}\right)\)


f^2(t1) = f(f(t1)) = f(2t1+t2) = 2 f(t1) + f(t2) = 2(2t1+t2) + (t2-2t3) = 4t1+3t2 - 2t3 (1.Spalte \(\tau D\tau^{2}\) )

also ist Deine Rechnung für die erste Spalte irgendwie korrekt. Nur ich würd entweder auf der Matrixseite oder Funktionsseite bleiben... vergleiche \(\tau D\tau^{2}\) )

usw.

Nach der Stelle

>"Und damit ist "

komm ich nicht mehr mit....

Avatar von 21 k

transponierte Matrizen wurden nicht behandelt. Ist mein Ansatz denn so auch richtig? Auch wenn es mehr Aufwand zu sein scheint

Wenn Du die Matrix gleich richtig aufscheibst, dann erübrig sich das transponieren....

Sorry: vor Fertigstellung zufällig auf Senden gekommen

Wenn Du die Matrix gleich richtig aufscheibst, dann erübrig sich das transponieren....

Heißt das nun mein Ansatz ist richtig, oder nicht?

Ich erhalte ja für die erste Spalte von \( D_T(f^2)\quad \begin{pmatrix} 4 \\ -9 \\ 5\end{pmatrix} \). Diese  erkenne ich in deiner Antwort ja nicht wieder...

Entschuldigung ich hab dummes Zeug gerechnet - Deine Matrix ist korrekt - ich überarbeite meine Antwort, damit das richtig gestellt ist.

Ich versuche Deinen Weg nachzuvollziehen - dauert einen Moment...

So, ich habs überarbeitet ich jetzt passt es- hoffentlich. Wie Du auf (4,-9,5) kommst ist mir aber nicht klar....

Einfach f(f(t2)) und f(f(t3)) durchrechnen wie bei f(f(t1)) gezeigt...

Nach der Stelle

>"Und damit ist "

komm ich nicht mehr mit....

Naja, ich habe ja das Bild \( f(2t_1+ t_2) =\begin{pmatrix} 2 \cdot 1 + 0 \\ 2 \cdot (-1) + (-1) \\ 2 \cdot 1 + 1 \end{pmatrix} =  f(\begin{pmatrix} 2 \\ -3 \\ 3\end{pmatrix}) \) mit der Darstellungsmatrix bezüglich der Basis T ermittelt. Da kam ich ja eine Zeile drüber auf \( \begin{pmatrix} 4 \\ 3 \\ -2\end{pmatrix} \) und das sind doch dann jeweils die Koeffizienten der Linearkombination von meiner Basis, also \(4t_1 + 3t_2 - 2t_3 = \begin{pmatrix} 4 \\ -9 \\ 5\end{pmatrix} \).

Aber wenn das so richtig ist, habe ich ja dann die Bestätigung die ich brauche, denn so wurde uns das in der Vorlesung beigebracht.


Vielleicht habe ich an deiner Antwort etwas nicht ganz verstanden, aber die erste Spalte\(\begin{pmatrix} 4 \\ -9 \\ 5\end{pmatrix} \) von meiner Lösung finde ich bei dir irgendwie wieder nicht auf.

Ich glaub ich versteht jetzt Deine Denke?

Also

f^2(t1)=f(f(t1))=(4 * t1) + (3 * t2) - (2 * t3 )

das sind die Koeffizienten der 1. Spalte der Ergebnismatrix - habe fertig - da ist nix weiter zu veranlassen - siehe 1.Spalte \(\tau D\tau^{2}\) . Du schreibst in die Matrix die Koeffizienten von der linearkombination {t1,t2,t3} auf der Grundlage der Baisis T - die koeffizienten von T also die Matrix die sich aus den Basisvektoren ergibt beschreibt einen Basiswechsel von T nach E

\(\epsilon T\tau \, :=  \, \left(\begin{array}{rrr}1&0&0\\-1&-1&1\\1&1&1\\\end{array}\right)\)

die spielen NICHT mit, weil wir in der Basis T unterwegs sind!

Weiter mit f^2(t2) und f^2(t3). Das Ergebnis muss sich in den Spalten 2,3 von \(\tau D\tau^{2}\) wiederfinden.

Die Hintereinander-Ausführung einer linearen Abb mit der Matrix A schreibt sich:

f(f(v)) = f^2(v) : A (A v) = A^2 v

Ach, na klar. Jetzt sehe ich wo mein Denkfehler war.

Vielen Dank für die Mühe und Geduld :)

Ein anderes Problem?

Stell deine Frage

Willkommen bei der Mathelounge! Stell deine Frage einfach und kostenlos

x
Made by a lovely community